LSAT and Law School Admissions Forum

Get expert LSAT preparation and law school admissions advice from PowerScore Test Preparation.

 Administrator
PowerScore Staff
  • PowerScore Staff
  • Posts: 8916
  • Joined: Feb 02, 2011
|
#25707
Complete Question Explanation

Parallel Reasoning—SN. The correct answer choice is (B)

This question also involves conditional reasoning, but thankfully is simpler than the previous question.

The argument centers around whether a specific 1996 ResearchTech project was funded by the government or by private corporations. We are told that, in 1996, all ResearchTech projects were funded in one of two ways, either they were funded by the government or by private corporations. This is a conditional relationship we can diagram as:

RT = ResearchTech project
G = funded by the government
C = funded by corporations

  • Sufficient ..... Necessary

    ..... ..... ..... ..... G
    RT1996 ..... :arrow: ..... or
    ..... ..... ..... ..... C
Next, we are told that the Gilman Survey, a 1996 ResearchTech project (RTGilman), was not funded by the government (G). Based on this evidence, the author concludes that the Gilman Survey must have been funded by private corporations (C).

This is a valid conclusion. The sufficient condition is satisfied, because the Gilman Survey was a 1996 ResearchTech project. Once the sufficient condition is satisfied, the necessary conditions must be met. Since we were told that it is was not a government project (G), the author rightly concluded that the funding must have been provided by private corporations (C).

The question stem tells us that this is a Parallel Reasoning question. We have a clear prephrase. The correct answer choice will contain a valid conditional argument, in which we are given a conditional rule with a compound necessary condition. The sufficient condition is satisfied, one of the two necessary conditions is ruled out, and the argument concludes the remaining necessary condition must be the case.

Answer choice (A): This choice begins by telling us that legal restrictions on consumer purchases have a “variety of aims.” The two options given, the purchases being either paternalistic or designed to protect civil liberties, are simply two examples of a potentially large set of options. This is unlike the stimulus, in which we are given exactly two options in an “or” compound necessary condition. In the stimulus, it was required to be one or the other. And, while the conclusion in this answer choice is definitive, as was the conclusion in the stimulus, the argument in the answer choice is flawed. Since the premises do not support the inference that it must be either paternalistic or for the protection of civil liberties, this argument is a False Dilemma.

Answer choice (B): This is the correct answer choice, because it matches the valid conditional argument presented in the stimulus. We can diagram the conditional rule as follows:

LR = legal restriction on consumer purchases
P = paternalistic
CL = protects civil liberties
  • ..... ..... ..... ..... ..... ..... P
    Premise: ..... LR ..... :arrow: ..... or
    ..... ..... ..... ..... ..... ..... CL
Next, we are told that Ordinance 304, a legal restriction on consumer purchases (LR), is not paternalistic (P). From this information, the argument concludes that Ordinance 304 must satisfy the only remaining necessary condition, it must be for the protection of civil liberties (CL).

Answer choice (C): This argument is flawed, because it concludes that neither necessary condition is satisfied, even though the sufficient condition has been met.

Answer choice (D): This argument contains two conditional rules, rather than the one rule we saw both in the stimulus and in the correct answer choice. The second rule is that “all ordinances passed in 1993 are paternalistic.” Even if this two rule structure were consistent with the stimulus, the argument is also flawed, because it contains a Mistaken Reversal. It concludes that, since a necessary condition (paternalistic) has been met, then the sufficient condition (legal restriction on consumer purchases) must be met.

Answer choice (E): This answer choice begins with a conditional principle, rather than a conditional statement of fact, and so can be eliminated very quickly. Remember that the correct answer choice in a Parallel Reasoning question must be entirely correct, with every element of the argument’s logical structure matching that of the stimulus. The benefit of our Parallel Reasoning Elemental Attack, in which we compare each logical element of the answer choice to the elements of the stimulus, is that you can accelerate through the answer choices by eliminating a choice, as we did here, as soon as any element fails to match. In that way, you can gain a competitive advantage, quickly slicing through this typically time-consuming question type.
 ronibass
  • Posts: 15
  • Joined: Jun 18, 2019
|
#71171
I am still confused about the difference between a conditional statement and a conditional principle. They both look the same to me. How can I distinguish them?
 James Finch
PowerScore Staff
  • PowerScore Staff
  • Posts: 943
  • Joined: Sep 06, 2017
|
#71309
Hi Roni,

The explanation is using "principle" to mean a guiding or moral principle, that something should be done when something else occurs, or in the case of (E), should be done only if one of two things occurs. This means that (E) can be instantly eliminated because the stimulus deals only with a statement of fact about the two possibilities for where research funding originated, rather that what should be done. Contrast this to the correct answer, (B), which deals with what is, rather than what should be.

Hope this clears things up!

Get the most out of your LSAT Prep Plus subscription.

Analyze and track your performance with our Testing and Analytics Package.